Use non-breaking space (~) between 'Figure' and the figure number.
[course.git] / latex / problems / Serway_and_Jewett_8 / problem23.62.tex
index eb338d3efeeb8a8e68ccf8df3c47e55c7c35a2d6..a3e67aabae5443a819435020516f2c4c9f4c498c 100644 (file)
@@ -1,6 +1,6 @@
 \begin{problem*}{23.62}
 Four identical charged particles ($q=+10.0\U{$\mu$C}$) are located on
-the corners of a rectangle as shown in Figure P23.62.  The dimensions
+the corners of a rectangle as shown in Figure~P23.62.  The dimensions
 of the rectangle are $L=60.0\U{cm}$ and $W=15.0\U{cm}$.
 Calculate \Part{a} the magnitude and \Part{b} the direction of the
 total electric force exerted on the charge at the lower left corner by